LSAT and Law School Admissions Forum

Get expert LSAT preparation and law school admissions advice from PowerScore Test Preparation.

 Administrator
PowerScore Staff
  • PowerScore Staff
  • Posts: 8924
  • Joined: Feb 02, 2011
|
#35309
Complete Question Explanation

StrengthenX. The correct answer choice is (C)

The psychiatrist’s argument only addresses the two extremes, in terms of student spending on recreation. The author says that students at both ends of this spectrum have similar anxiety and depression scores. However, the author does not address the anxiety and depression scores of students in the middle of the range, and the conclusion is vulnerable to criticism that if the high spending reduce their spending to only a moderate level, we have no data to predict what their anxiety and depression scores may be. Since this a Strengthen—Except question, we can expect that the four incorrect answers will strengthen the argument by addressing this flaw. The correct answer, however, will either have no effect on the argument or will weaken it.

Answer choice (A): Pay close attention to the scope of the conclusion. The author is predicting what may happen to first-year students generally. Thus, evidence that the results at other universities mirror those at this university would help the argument by suggesting that the results are more likely generalizable. Answer choice (A) is thus incorrect.

Answer choice (B): This is an important issue to resolve when assessing the author’s conclusion. The author claims that the highest-spending students can “reduce that spending [on recreation] without increasing their anxiety or depression.” For this to be true, it must be the case that both significant and minor decreases in spending would not be associated with increased anxiety or depression. (B) strengthens this claim by showing that neither moderate nor low spenders have higher anxiety scores than high spenders.

Answer choice (C): This is the correct answer choice. If (C) is true, reduced spending is strongly correlated with increased levels of anxiety and depression. This suggests that the best way to avoid increasing anxiety or depression is have an increased level of spending on recreation. Also, conclusions about 40- to 60-year-old adults may not be representative of first-year student at universities, as significant differences in lifestyle, disposable income, and recreation choices may make direct comparisons difficult. Since (C) will, at best, be irrelevant and, at worst, damage the conclusion, this answer does not strengthen.

Answer choice (D): Evidence that the screening instruments are accurate helps justify their use in drawing conclusions about first-year students. Answer choice (D) strengthens the argument.

Answer choice (E): Answer choice (E) is clearly helpful to the argument, as it demonstrates the psychiatrist’s recommendation has a practical basis, as well as a statistical rationale. Since the psychiatrist’s patients are also first-year students at the university, there is good reason that their experience is representative of the students referred to in the conclusion.
 Haleyeastham
  • Posts: 33
  • Joined: Aug 03, 2015
|
#19950
Can you please explain correct answer choice c to me?

Thanks!
 Jon Denning
PowerScore Staff
  • PowerScore Staff
  • Posts: 904
  • Joined: Apr 11, 2011
|
#19953
Hey Haley,

I'm happy to help you out! However, if possible, it would be really useful to hear exactly how worked through this question!

That is, each of the following would be great to know: what did you think of the stimulus when you read it? Did you identify the conclusion, and if so what do you think it is? Did you see any reasoning issues (reasons to be suspicious of the argument)? What kind of question did you believe this to be, and what strategies did you then anticipate being useful in finding an answer (tell me what you know about this question type and the right vs wrong answers)? What was your prephrase? Which answer or answers gave you trouble—which did you pick and why, and what was it about the correct answer you found unappealing?

Knowing the specifics of how you interpreted the question and the things you did right and wrong will help us a ton as we try to provide you with an explanation that addresses your situation as closely as possible :)

Thanks in advance and I look forward to hearing from you!

Jon
 lsat2016
  • Posts: 59
  • Joined: May 29, 2016
|
#28012
How does C affect the argument?

The conclusion is saying that high recreation spenders can reduce that spending without increasing anxiety.

Answer C is saying that high recreation spenders are strongly correlated with decreased levels of anxiety. Doesn’t this strengthen the argument?
 Emily Haney-Caron
PowerScore Staff
  • PowerScore Staff
  • Posts: 577
  • Joined: Jan 12, 2012
|
#28042
Hi lsat2016,

Answer C actually doesn't impact the argument either way, because it is talking about a completely different population. There's no reason to think first-year college students and adults aged 40-60 would be the same. Does that help?
 mokkyukkyu
  • Posts: 97
  • Joined: Aug 17, 2016
|
#28160
Hi,

I don't understand how B helps the argument...
could anyone explain how the information of "moderate students" helps in this case?
 Emily Haney-Caron
PowerScore Staff
  • PowerScore Staff
  • Posts: 577
  • Joined: Jan 12, 2012
|
#28179
Hi mokkyukkyu,

B helps because it is essentially saying that a moderate level of spending is best in terms of anxiety and depression (i.e., better than highest or lowest levels). If that is true, then students with high levels of spending who drop them spending to a moderate level not only won't increase their anxiety and depression, but might actually decrease anxiety and depression.

Hope that helps!
 deck1134
  • Posts: 160
  • Joined: Jun 11, 2018
|
#49360
Hi PowerScore,

I immediately trashed answer (C) because of the age difference, that it was too great to yield a similarity. Should I have also seen the "spending" issue with it?

Thanks.
User avatar
 Dave Killoran
PowerScore Staff
  • PowerScore Staff
  • Posts: 5853
  • Joined: Mar 25, 2011
|
#50069
deck1134 wrote:Hi PowerScore,

I immediately trashed answer (C) because of the age difference, that it was too great to yield a similarity. Should I have also seen the "spending" issue with it?

Thanks.
No, because spending is a major part of the stimulus. There'd be no reasonable expectation that an answer with spending would be irrelevant, and indeed you can see that (A), (B), and (E) each mention spending. The test makers likely threw spending into (C) precisely to make it more attractive.

Thanks!

Get the most out of your LSAT Prep Plus subscription.

Analyze and track your performance with our Testing and Analytics Package.